A set F is closed if and only if every convergent sequence in F converges to a limit in F











up vote
0
down vote

favorite












$F subseteq X$, where $X$ is a metric space.



Then $F$ is closed if and only if every convergent sequence in F converges to a limit in $F$.



Attempt



$ implies$ If $F$ is closed, $F= overline F$, but $overline F=F cup F'$ where $F'$ is the set of all accumulation points of $F$.



Thus, $F' subseteq F$ which means that $F$ contains all of its limit points which means that every sequence in F converges to a limit that is in F.



Is this a valid proof for this direction?



Edit
After reading the comments, I should have written every convergent sequence in F converges to a limit in F.










share|cite|improve this question
























  • You may find your answer here: math.stackexchange.com/questions/882876/…
    – Aniruddha Deshmukh
    Nov 29 at 4:20






  • 1




    If F is the reals, your conclusion that every sequence within F converges to a limit of F, is false.
    – William Elliot
    Nov 29 at 4:26










  • So if F contains all its limit points, it does not imply that every sequence in F converges to a limit in F?
    – Snop D.
    Nov 29 at 4:31






  • 1




    @SnopD.: You can say for every point in $F'$ there exist a sequence in $F$ such that it converges to that point.
    – Yadati Kiran
    Nov 29 at 4:36

















up vote
0
down vote

favorite












$F subseteq X$, where $X$ is a metric space.



Then $F$ is closed if and only if every convergent sequence in F converges to a limit in $F$.



Attempt



$ implies$ If $F$ is closed, $F= overline F$, but $overline F=F cup F'$ where $F'$ is the set of all accumulation points of $F$.



Thus, $F' subseteq F$ which means that $F$ contains all of its limit points which means that every sequence in F converges to a limit that is in F.



Is this a valid proof for this direction?



Edit
After reading the comments, I should have written every convergent sequence in F converges to a limit in F.










share|cite|improve this question
























  • You may find your answer here: math.stackexchange.com/questions/882876/…
    – Aniruddha Deshmukh
    Nov 29 at 4:20






  • 1




    If F is the reals, your conclusion that every sequence within F converges to a limit of F, is false.
    – William Elliot
    Nov 29 at 4:26










  • So if F contains all its limit points, it does not imply that every sequence in F converges to a limit in F?
    – Snop D.
    Nov 29 at 4:31






  • 1




    @SnopD.: You can say for every point in $F'$ there exist a sequence in $F$ such that it converges to that point.
    – Yadati Kiran
    Nov 29 at 4:36















up vote
0
down vote

favorite









up vote
0
down vote

favorite











$F subseteq X$, where $X$ is a metric space.



Then $F$ is closed if and only if every convergent sequence in F converges to a limit in $F$.



Attempt



$ implies$ If $F$ is closed, $F= overline F$, but $overline F=F cup F'$ where $F'$ is the set of all accumulation points of $F$.



Thus, $F' subseteq F$ which means that $F$ contains all of its limit points which means that every sequence in F converges to a limit that is in F.



Is this a valid proof for this direction?



Edit
After reading the comments, I should have written every convergent sequence in F converges to a limit in F.










share|cite|improve this question















$F subseteq X$, where $X$ is a metric space.



Then $F$ is closed if and only if every convergent sequence in F converges to a limit in $F$.



Attempt



$ implies$ If $F$ is closed, $F= overline F$, but $overline F=F cup F'$ where $F'$ is the set of all accumulation points of $F$.



Thus, $F' subseteq F$ which means that $F$ contains all of its limit points which means that every sequence in F converges to a limit that is in F.



Is this a valid proof for this direction?



Edit
After reading the comments, I should have written every convergent sequence in F converges to a limit in F.







real-analysis proof-verification metric-spaces






share|cite|improve this question















share|cite|improve this question













share|cite|improve this question




share|cite|improve this question








edited Nov 29 at 5:01

























asked Nov 29 at 4:16









Snop D.

285




285












  • You may find your answer here: math.stackexchange.com/questions/882876/…
    – Aniruddha Deshmukh
    Nov 29 at 4:20






  • 1




    If F is the reals, your conclusion that every sequence within F converges to a limit of F, is false.
    – William Elliot
    Nov 29 at 4:26










  • So if F contains all its limit points, it does not imply that every sequence in F converges to a limit in F?
    – Snop D.
    Nov 29 at 4:31






  • 1




    @SnopD.: You can say for every point in $F'$ there exist a sequence in $F$ such that it converges to that point.
    – Yadati Kiran
    Nov 29 at 4:36




















  • You may find your answer here: math.stackexchange.com/questions/882876/…
    – Aniruddha Deshmukh
    Nov 29 at 4:20






  • 1




    If F is the reals, your conclusion that every sequence within F converges to a limit of F, is false.
    – William Elliot
    Nov 29 at 4:26










  • So if F contains all its limit points, it does not imply that every sequence in F converges to a limit in F?
    – Snop D.
    Nov 29 at 4:31






  • 1




    @SnopD.: You can say for every point in $F'$ there exist a sequence in $F$ such that it converges to that point.
    – Yadati Kiran
    Nov 29 at 4:36


















You may find your answer here: math.stackexchange.com/questions/882876/…
– Aniruddha Deshmukh
Nov 29 at 4:20




You may find your answer here: math.stackexchange.com/questions/882876/…
– Aniruddha Deshmukh
Nov 29 at 4:20




1




1




If F is the reals, your conclusion that every sequence within F converges to a limit of F, is false.
– William Elliot
Nov 29 at 4:26




If F is the reals, your conclusion that every sequence within F converges to a limit of F, is false.
– William Elliot
Nov 29 at 4:26












So if F contains all its limit points, it does not imply that every sequence in F converges to a limit in F?
– Snop D.
Nov 29 at 4:31




So if F contains all its limit points, it does not imply that every sequence in F converges to a limit in F?
– Snop D.
Nov 29 at 4:31




1




1




@SnopD.: You can say for every point in $F'$ there exist a sequence in $F$ such that it converges to that point.
– Yadati Kiran
Nov 29 at 4:36






@SnopD.: You can say for every point in $F'$ there exist a sequence in $F$ such that it converges to that point.
– Yadati Kiran
Nov 29 at 4:36












1 Answer
1






active

oldest

votes

















up vote
2
down vote



accepted










That's a valid proof (but you wrote every sequence where you should have wrote every convergent sequence).



For the converse:
If every convergent sequence in $A$ converges to a point in $A$, then $A'subset A$. Thus A is closed.






share|cite|improve this answer





















    Your Answer





    StackExchange.ifUsing("editor", function () {
    return StackExchange.using("mathjaxEditing", function () {
    StackExchange.MarkdownEditor.creationCallbacks.add(function (editor, postfix) {
    StackExchange.mathjaxEditing.prepareWmdForMathJax(editor, postfix, [["$", "$"], ["\\(","\\)"]]);
    });
    });
    }, "mathjax-editing");

    StackExchange.ready(function() {
    var channelOptions = {
    tags: "".split(" "),
    id: "69"
    };
    initTagRenderer("".split(" "), "".split(" "), channelOptions);

    StackExchange.using("externalEditor", function() {
    // Have to fire editor after snippets, if snippets enabled
    if (StackExchange.settings.snippets.snippetsEnabled) {
    StackExchange.using("snippets", function() {
    createEditor();
    });
    }
    else {
    createEditor();
    }
    });

    function createEditor() {
    StackExchange.prepareEditor({
    heartbeatType: 'answer',
    autoActivateHeartbeat: false,
    convertImagesToLinks: true,
    noModals: true,
    showLowRepImageUploadWarning: true,
    reputationToPostImages: 10,
    bindNavPrevention: true,
    postfix: "",
    imageUploader: {
    brandingHtml: "Powered by u003ca class="icon-imgur-white" href="https://imgur.com/"u003eu003c/au003e",
    contentPolicyHtml: "User contributions licensed under u003ca href="https://creativecommons.org/licenses/by-sa/3.0/"u003ecc by-sa 3.0 with attribution requiredu003c/au003e u003ca href="https://stackoverflow.com/legal/content-policy"u003e(content policy)u003c/au003e",
    allowUrls: true
    },
    noCode: true, onDemand: true,
    discardSelector: ".discard-answer"
    ,immediatelyShowMarkdownHelp:true
    });


    }
    });














    draft saved

    draft discarded


















    StackExchange.ready(
    function () {
    StackExchange.openid.initPostLogin('.new-post-login', 'https%3a%2f%2fmath.stackexchange.com%2fquestions%2f3018172%2fa-set-f-is-closed-if-and-only-if-every-convergent-sequence-in-f-converges-to-a-l%23new-answer', 'question_page');
    }
    );

    Post as a guest















    Required, but never shown

























    1 Answer
    1






    active

    oldest

    votes








    1 Answer
    1






    active

    oldest

    votes









    active

    oldest

    votes






    active

    oldest

    votes








    up vote
    2
    down vote



    accepted










    That's a valid proof (but you wrote every sequence where you should have wrote every convergent sequence).



    For the converse:
    If every convergent sequence in $A$ converges to a point in $A$, then $A'subset A$. Thus A is closed.






    share|cite|improve this answer

























      up vote
      2
      down vote



      accepted










      That's a valid proof (but you wrote every sequence where you should have wrote every convergent sequence).



      For the converse:
      If every convergent sequence in $A$ converges to a point in $A$, then $A'subset A$. Thus A is closed.






      share|cite|improve this answer























        up vote
        2
        down vote



        accepted







        up vote
        2
        down vote



        accepted






        That's a valid proof (but you wrote every sequence where you should have wrote every convergent sequence).



        For the converse:
        If every convergent sequence in $A$ converges to a point in $A$, then $A'subset A$. Thus A is closed.






        share|cite|improve this answer












        That's a valid proof (but you wrote every sequence where you should have wrote every convergent sequence).



        For the converse:
        If every convergent sequence in $A$ converges to a point in $A$, then $A'subset A$. Thus A is closed.







        share|cite|improve this answer












        share|cite|improve this answer



        share|cite|improve this answer










        answered Nov 29 at 4:38









        Chris Custer

        10.4k3724




        10.4k3724






























            draft saved

            draft discarded




















































            Thanks for contributing an answer to Mathematics Stack Exchange!


            • Please be sure to answer the question. Provide details and share your research!

            But avoid



            • Asking for help, clarification, or responding to other answers.

            • Making statements based on opinion; back them up with references or personal experience.


            Use MathJax to format equations. MathJax reference.


            To learn more, see our tips on writing great answers.





            Some of your past answers have not been well-received, and you're in danger of being blocked from answering.


            Please pay close attention to the following guidance:


            • Please be sure to answer the question. Provide details and share your research!

            But avoid



            • Asking for help, clarification, or responding to other answers.

            • Making statements based on opinion; back them up with references or personal experience.


            To learn more, see our tips on writing great answers.




            draft saved


            draft discarded














            StackExchange.ready(
            function () {
            StackExchange.openid.initPostLogin('.new-post-login', 'https%3a%2f%2fmath.stackexchange.com%2fquestions%2f3018172%2fa-set-f-is-closed-if-and-only-if-every-convergent-sequence-in-f-converges-to-a-l%23new-answer', 'question_page');
            }
            );

            Post as a guest















            Required, but never shown





















































            Required, but never shown














            Required, but never shown












            Required, but never shown







            Required, but never shown

































            Required, but never shown














            Required, but never shown












            Required, but never shown







            Required, but never shown







            Popular posts from this blog

            Different font size/position of beamer's navigation symbols template's content depending on regular/plain...

            Berounka

            I want to find a topological embedding $f : X rightarrow Y$ and $g: Y rightarrow X$, yet $X$ is not...